Psychiatrist: In treating first-year students at this university, I have noticed that those reporting the highest le...

LizD on September 4, 2018

trouble with the except Questons

Hello. I was hoping someone could walk me through this question? Thank you.

Reply
Create a free account to read and take part in forum discussions.

Already have an account? log in

Max-Youngquist on September 5, 2018

@lizd for a "all these strengthen EXCEPT" question, look for two types of answer choices:

1) an answer that weakens the argument
2) an answer that has no effect on the argument

Conclusion: students w the HIGHEST levels of spending can REDUCE spending WITHOUT INCREASING anxiety/depression

So looking at the choices:

(A) strengthens the argument, because if those w the lowest spending have the same levels of A/D as those with the highest spending, then there's no evidence to suggest A/D would increase if spending was lowered

(B) strengthens the argument. If those with moderate spending have lowest A/D, than those w the highest spending could reduce A/D by reducing spending

(C) is the correct answer because it has NO EFFECT on the argument (i.e. it does not strengthen it). Remember, this argument was only ever talking about first year students, whereas (C) is talking about 40-60 year olds. We don't care what happens to 40 - 60 year olds (although even if you missed that, you could argue the conclusion about 40 - 60 year olds would weaken the argument because it states that A/D decreases as spending goes up, so reducing spending should increase A/D).

(D) strengthens the argument, in that it gives us more evidence to suggest the observations of A/D made were accurate

(E) strengthens the argument by giving some specific examples of students who match the psychiatrist's conclusion

I hope that helps!